Which of the subsets of R3 is a subspace of R3?(a) W =

Chapter 4, Problem 4.25

(choose chapter or problem)

Which of the subsets of \(R^{3}\) is a subspace of \(R^{3}\)?

(a) \(W=\left\{\left(x_{1}, x_{2}, x_{3}\right): x_{1}^{2}+x_{2}^{2}+x_{3}^{2}=0\right\}\)

(b) \(W=\left\{\left(x_{1}, x_{2}, x_{3}\right): x_{1}^{2}+x_{2}^{2}+x_{3}^{2}=1\right\}\)

Text Transcription:

R^3

R^3

W={(x_1, x_2, x_3): x_1 ^2 +x_2 ^2 +x_3 ^2 =0}

W={(x_1, x_2, x_3): x_1 ^2 +x_2 ^2 +x_3 ^2 =1}

Unfortunately, we don't have that question answered yet. But you can get it answered in just 5 hours by Logging in or Becoming a subscriber.

Becoming a subscriber
Or look for another answer

×

Login

Login or Sign up for access to all of our study tools and educational content!

Forgot password?
Register Now

×

Register

Sign up for access to all content on our site!

Or login if you already have an account

×

Reset password

If you have an active account we’ll send you an e-mail for password recovery

Or login if you have your password back